LSAT and Law School Admissions Forum

Get expert LSAT preparation and law school admissions advice from PowerScore Test Preparation.

 Administrator
PowerScore Staff
  • PowerScore Staff
  • Posts: 8927
  • Joined: Feb 02, 2011
|
#26549
Complete Question Explanation
(The complete setup for this game can be found here: lsat/viewtopic.php?t=11086)

The correct answer choice is (B)

Answer choice (A) is incorrect because M cannot be ranked last.

Answer choice (C) is incorrect because if J is ranked third, then from the fourth rule H must be ranked first, a violation of the second rule.

Answer choice (D) is incorrect because when K is ranked second then L must be ranked fifth, and when H is ranked third, then J must be ranked fifth, and according to the first rule there are no ties.

Answer choice (E) is incorrect because there is no available ranking for O when M is seventh and L is eighth.

Accordingly, answer choice (B) is proven correct by elimination.
User avatar
 lleisure
  • Posts: 1
  • Joined: Jun 24, 2021
|
#88257
Hello!

I was wondering if someone could help me with a more complete breakdown or step-by-step explanation of how I would determine that answer choices C and D are wrong. I'm having a hard time envisioning how I start to go about trying different orders of variables outside of the obvious and easy thought process like "answer choice A must be wrong because there is an M not law."

Thank you!! :)
 Robert Carroll
PowerScore Staff
  • PowerScore Staff
  • Posts: 1774
  • Joined: Dec 06, 2013
|
#88307
lleisure,

It might be a little difficult to see from the pure text how answer choices (C) and (D) don't work, so I'd suggest writing them down and making the inferences they induce.

Answer choice (C): Once we put J third, we should immediately think about H, which is an exact distance before J, given the 4th rule. This answer places H first. But on your main diagram, position 1 will always be F or G, thus not H, so this answer is out - it doesn't matter if the rest of it could happen, the first half is already impossible.

Answer choice (D): Put K second and H third. We should immediately think of where J and L go, as they are a fixed distance from H and K respectively. The problem is that J will be fifth, as will L, given the 4th and 5th rules, so this answer does not work.

Robert Carroll

Get the most out of your LSAT Prep Plus subscription.

Analyze and track your performance with our Testing and Analytics Package.